Sei sulla pagina 1di 18

DC NETWORK THEOREM

Current: “Rate of flow of electric charge.”


Charge
I= Coulombs/Sec or Ampere
time
Note:-
1. Direction of current is same as the direction of motion of +Ve charge or opposite to the direction of
motion of –Ve charge.

Voltage: “Energy required in transferring a charge of one coulomb from one point to
another point.”
Energy(W)
V= Joule/Coulomb or Volts
Charge(Q)

EMF (Electromotive force): “The EMF of a voltage source is the energy imparted by
the source to each coulomb of the charge passing through it.”
Energy(W)
E= Joule/Coulomb or Volts
Charge(Q)

Potential Difference: “The pd between two points is the energy required in transferring a
charge of coulomb from one point to another point.”
Energy(W)
pd = Joule/Coulomb or Volts
Charge(Q)

Voltage drop: “The voltage drop between two points is the decrease in energy required
in transferring a charge of coulomb from one point to another point.”
Energy(W)
Voltage drop = Joule/Coulomb or Volts
Charge(Q)

Resistance: “Electric resistance is the property of material which offers opposition to the
flow of current and dissipates energy.”
l
R=ρ Ohm or Ω (Law of resistance)
a
Where l = Length of the wire
a = cross − sectional area of the wire
ρ = Resistivit y or Specific Resistance of the material
Note:-
1. Resistance also depends on temperature.

Ohm’s Law: “The current passing through a conductor is directly proportional to


potential drop across its ends provided physical conditions are same.”
I ∞V
1
Or I= V
R
Or V = IR
1 Prepared by: Nafees Ahmed www.eedofdit.webnode.com
Conductance (G): It is reciprocal of resistance

1 a a
G= = =σ mho, Ω −1 , Siemens
Resistance (R) ρl l

Power & Energy in an electric circuit:


Power P = VI Watt, KW, MW
= I 2R
V2
=
R
Energy E = P × time Joule, watt-sec, watt-hours, KWH

Series Circuit:
R1 R2 R3 Req

I V1 V2 V3 ⇒ I
V V
(b)
(a)
Fig.1

We know V1 = IR1 V2 = IR2 V3 = IR3

Total voltage V = V1 + V 2 + V 3
⇒ IReq = IR1 + IR2 + IR3
⇒ Req = R1 + R2 + R3
Note:-
1. Same current means resistances are in series.

Example 1: Three resistors are connected in series across a 12V battery. The one
resistance has a value of 1 ohm, second has a voltage drop of 4 Volts & third has power
dissipation of 12 W. Calculate value of each resistance & circuit current.
Solution: Hint V = V1 + V2 + V3
12 = I + 4 + 12 / I
I = 2 Or 6 Amperes
When I=2 Amp
R1 = 1 R2 = 2 R3 = 3
When I=6 Amp
R1 = 1 R2 = 2/3 R3 = 1/3

2 Prepared by: Nafees Ahmed www.eedofdit.webnode.com


Parallel Circuit:
I1 R1
Req
I2 R2
⇒ I
V
I3 R2
(b)
I
V
(a) Fig.2
V V V
We know I1 = I2 = I3 =
R1 R2 R3
Total current I = I1 + I 2 + I 3
V V V V
⇒ = + +
Req R1 R2 R3
1 1 1 1
⇒ = + +
Req R1 R2 R3
Note:-
1. Same voltage means resistances are in parallel.
2. If two resistances are in parallel and R1=R2=R then
R
Req =
2

Series Parallel circuit:

I4 R4
I1 R1
I3 R3 I5 R5
I2 R2 B D
C
A I6 R6 RAD
A RAB B RBC C RCD D
I VAB VBC VCD I
I V
V V
(a) (b) (C)
(c)
Fig.3
R1R2 1 1 1 1
RAB = RBC = R3 = + +
R1 + R2 RCD R4 R5 R6
V
So RAD = RAB + RBC + RCD And I =
RAD
VAB = IRAB From fig.3b
VBC = IRBC From fig.3b
VCD = IRCD From fig.3b

3 Prepared by: Nafees Ahmed www.eedofdit.webnode.com


VAB VAB VBC
I1 = I2 = I3 = =I From fig.3a
R1 R2 R3

VCD V V
I4 = I 5 = CD I 6 = CD From fig.3a
R4 R5 R6
Power consumed by whole circuit
P = I 2 RAD From fig.3c
Or = I R AB + I R BC + I R CD
2 2 2
From fig.3b
Or = I12 R1 + I 22 R2 + I 32 R3 + I 42 R4 + I 52 R5 + I 62 R6 From fig.3a

Example 2: Two resistances of 20 & 30 ohms respectively are connected in parallel.


These two parallel resistances are further connected in series with a third resistance of 15
ohm. If current through 15 ohm resistance is 3 Amperes. Find
(i) Current through 20 & 30 ohm resistances
(ii) Voltage across whole circuit I1 20
(iii) Total power consumed. 3 A 15
Solution: Hint I2 30
B C
Req = (20||30) + 15 A
= 12 + 15 =37
V AB = 12x3 = 36 I
(i) I 1 = 36 / 20 = 1.8 A I 2 = 36 / 30 = 1.2 A V
(ii) V AC = V AB + V BC = 36 + 15 × 3 = 81Volts Fig. 4
(iii) Power = VI = 243 Watts

Circuit or Network Elements: R, L, C

Network or Circuit: “Path followed by an electric current.”


Or
“Any interconnection of circuit elements with or without energy sources.”
Note:-
1. A circuit must have a closed energized path.
2. A network may not have a closed path i.e. T-Network

Fig.5-T-Network
3. So every network may not be circuit (i.e. T-Network) but every circuit is a network.

Loop: “Any closed path in the network.”

Mesh: “A closed path which does not have any closed path inside it.”

Node: “It is a junction where 2 or more branches are connected together.”

4 Prepared by: Nafees Ahmed www.eedofdit.webnode.com


Note:-
1. A junction where 3 or more branches connected together is known as principal node or
essential node.
2. For example A B C
Loop but not Mesh
H

D
F E
Loop or Mesh Loop or Mesh
Fig.6
Total no of nodes = 6+2 = 8 (A, B, C, D, E, F, G, H)
No of principal nodes =2 (B & E)
No of Loops =3 (ABEFA, BCDEB, ABCDEFA)
No of Meshes =2 (ABEFA, BCDEB)

Energy Sources:
Basically Two types
(A) Independent Energy Sources
a. Voltage Source
i. AC/DC
ii. Ideal/Practical
b. Current Source
i. AC/DC
ii. Ideal/Practical
(B) Dependent Energy Sources
a. Voltage Source
b. Current Source

(A) Independent Energy Sources:


a. Voltage Source

r
V V

(a) Ideal DC Voltage Source (b) Practical DC Voltage Source


(Internal Resistance, r=0) (With Internal Resistance, r)

r
+
V +
_ V_

(c)Ideal AC Voltage Source


(Internal Resistance, r=0) Fig.7 (d) Ideal AC Voltage Source
(Internal Resistance, r)

5 Prepared by: Nafees Ahmed www.eedofdit.webnode.com


b. Current Source

I I R

(a) Ideal Current Source (b) Practical Current Source


(Internal Resistance, R=infinity) (Internal Resistance, R)
Fig.8

(B) Dependent Energy Sources I1 R1


Consider some branch of a circuit
V1

+ bI1 +
_
aV1 _
Volts Volts

(a) Voltage Dependent Voltage (b) Current Dependent Voltage


Source (VDVS) Source (CDVS)

cV1 dI1
Amperes Amperes

(c) Voltage Dependent Current (d) Current Dependent Current


Source (VDCS) Source (CDCS)
Fig.9

Kirchhoff’s Current Law (KCL):


“At any instant of time, the algebraic sum of currents at any node is zero.”
∑I = 0
Or
“Total incoming currents = Total outgoing currents.”

Kirchhoff’s Voltage Law (KVL):


“At any instant of time, the algebraic sum of voltages in a closed path is zero.”

6 Prepared by: Nafees Ahmed www.eedofdit.webnode.com


Consider following circuit

I1 R1 R2
A B I2 C

V2
R6 R3
I3 R4
V1
D
F E
Fig.10

Apply KCL at node B I1 - I2 - I3 = 0 Or I1 = I2 + I3


Apply KVL in loop ABEFA
I 1 R 1 + I 3 R3 − V1 + I 1 R6 = 0
Apply KVL in loop BCDEB
I 2 R 2 − V 2 + I 2 R 4 − I 3 R3 = 0
Apply KVL in loop ABCDEFA
I 1 R1 + I 2 R2 − V2 + I 2 R4 − V1 + I 2 R6 = 0

Example 3: In the following circuit calculate current in each branch by KCL & KVL
(Fig.11a).
Solution:
B I1+I2
A C
I1 I2
2Ω 4Ω 2Ω 4Ω
6Ω 6Ω
40V 44V 40V 44V
F E D
(a)
(b)
Fig.11
Apply KVL in loop ABEFA − 40 + 2 I 1 − 4 I 2 + 44 = 0 − − − − − − − −(1)
Apply KVL in loop BCDEB − 44 + 4 I 2 + 6( I 1 + I 2 ) = 0 − − − − − − − (2)
34 28 62
Solving (1) & (2) I1 = A I2 = A I1 + I 2 = A
11 11 11
Mesh Analysis:
Steps are
1. Identify the total meshes
2. Assume some mesh current in each mesh (clockwise or anticlockwise)
3. Apply KVL in each mesh.
4. Solve the above equations.

7 Prepared by: Nafees Ahmed www.eedofdit.webnode.com


Example 4: Solve Example 3 by Mesh Analysis.
Solution:
Let the two meshes are having clockwise currents as shown in following figure

Apply KVL in loop 1 4Ω


− 40 + 2 I 1 − 4( I 1 − I 2 ) + 44 = 0 − − − −(1) 2Ω
Apply KVL in loop 2 I1 I2
6Ω
− 44 + 4( I 2 − I 1 ) + 6 I 2 = 0 − − − −(2) 40V
Solving (1) & (2) 44V
34 28 62 Fig.12
I1 = A I 2 − I1 = A I2 = A
11 11 11

Example 5: Find the current through R3=4 ohm resistance by mesh analysis (Fig.13a)
Solution:

2Ω 2 2 R3
2Ω R3
10V 5A 10V I1 I2 I3
5A

(a) Fig.13 (b)

Let the three meshes are having clockwise currents as shown in following figure
Apply KVL in mesh 1
− 10 + 2 I1 + 2( I1 − I 2 ) = 0 − − − − − −(1)
Apply KVL in mesh 2
+ 2( I 2 − I1 ) + 4( I 2 − I 3 ) = 0 − − − − − −(2)
Clearly From mesh 3
I 3 = −5 A − − − − − −(3)
Solving (1), (2) & (3) I 1 = −1 A I 2 = −3
Current in R3 = I2-I3 = (-3)-(-5) = 2 A

Node Analysis:
Steps are
1. Identify the total Principal Nodes
2. Assume one node as reference node (Voltage of this node = 0 Volts)
3. Assume some node voltages for other remaining nodes w.r.to
reference node. (V1, V2, V3 etc).
4. Assume some branch currents in different branches.
5. Apply KCL at different nodes and make the equations in terms of node
voltages and circuit elements.
6. Solve the above equations.

8 Prepared by: Nafees Ahmed www.eedofdit.webnode.com


Example 6: Solve Example 3 by Node Analysis.
Solution:
Apply KCL at node V1

 V1 − 40 − 0   V1 − 44 − 0   V1 − 0 
 + + =0
 2   4   6 
372 I1 V1 I3
V1 = Volts I2
11
34 2 4
So I1 = − A
11 6
40V 44V
28
I2 = − A Reference or
11
Fig.14 Datum Node, V=0
62
I3 = A
11

Example 7: Solve Example 5 by Node Analysis. I1 V1 I4


Solution: 2Ω I2 I3
Apply KCL at node V1 2Ω R3
10V
5A
 V1 − 10 − 0   V1 − 0   V1 − 0 
 + + =5 V=0
 2   2   4  Fig.15
V1 = 8 Volts
V − 0 
So I3 =  1  = 2 Amps
 4 
Superposition theorem:
“In a linear circuit, containing more then one independent
energy sources, the overall response (Voltage or current) in any branch of the circuit is
equal to sum of the response due to each independent source acting one at a time while
making other source in-operative.”

I I’ I”
R1 R1 + R1
I1 ⇒
R2 R2 R2
I1
V1 V1

(a) (b) (c)

Fig.16

So according to superposition theorem I=I’+I”

9 Prepared by: Nafees Ahmed www.eedofdit.webnode.com


Example 8: Solve example 5 by superposition theorem.
Solution:

2 I’ 2 I”
2 R3 2 R3
10V 5A

(a) (b)
Fig.17
Consider 10 V Voltage source only (Fig.17a)
 2  10 
I'=    = 1 Amps
 2 + 4  2 + (2 || 4) 

Consider 5A Current Source only (Fig.17b)


 1 
I" =  5 = 1 Amps
1 + 4 
So I=I’+I”=1+1=2 Amps

Example 9: Find I in the following circuit by superposition theorem (Fig.18a).


Solution:
5 5 I1
I Ω I’
20A 20A
8 4 12 8 4 12
10V 40 V
(a) (b)
Consider 20A current source only (Fig.18b)
5 I2
By current division rule
 8  I”
I 1 =  20 = 10 8 4 12
 8 + 5 + (4 || 12)  Ω
Again bye current division rule 10V
 4  5 (c)
I'=  10 = Amps
 4 + 12  2
5 V1
Consider 10V Voltage source only (Fig.18c)
I”
8 4 ’12
 10  5
I 2 = −  = − 40V
 8 + 5 + (4 || 12)  8
By current division rule
(d)
 4  5  5 Fig.18
I" =   −  = − Amps
 4 + 12  8  32

10 Prepared by: Nafees Ahmed www.eedofdit.webnode.com


Consider 40V Voltage source only (Fig.18d)

Apply KCL at node V1


 V1 − 0   V1 − 40 − 0   V1 − 0 
 + = 
 8+5   4   12 
6240
V1 = V
256
 V − 0  65
I '''=  1  = Amps
 12  32
 5   5   65  35
So I = I'+ I' '+ I' ' ' =   +  −  +   = Amps
 2   8   32  8

Thevenin’s theorem:
“Any linear two terminal circuits can be replaced by an
equivalent network consisting of a voltage source (VTh) in series with a resistance (RTh).”

A A
Any Linear
IL RTh IL
Two
RL ⇒ RL
terminals
VTh
Network
B B
(b) Thevenin’s Equivalent
(a)
Circuit
Fig.19

Where
VTh = Open circuit voltage at load terminals
RTh = Equivalent resistance of the network at load terminals when the
sources are made in-operative.
VTh
And IL =
RTh + RL

Example 10: Solve example 5 by Thevenin’s Theorem.


Solution:
VTh
2 2
2 Ω 2 VTh
RTh
10V Ω 5
A
V=0
(a)
(b)
Fig.20

11 Prepared by: Nafees Ahmed www.eedofdit.webnode.com


Find RTh (Fig.20a)
RTh = (2 || 2) = 1Ω

Find VTh (Fig.20b)


Apply KCL at node VTh A
RTh IL
 VTh − 10 − 0   VTh − 0  RL
 + =5
 2   2  VTh
VTh = 10 Volts
B
(c) Thevenin’s Equivalent
VTh 10 Circuit
So I L = = = 2 Amps
RTh + RL 1 + 4

Example 11: Find Thevenin’s equivalent circuit across AB of following circuit


(Fig.21a).
Solution:
4A

A A
20 Ω 3Ω 20 Ω 3Ω
5Ω 5Ω RTh
30V
2Ω B 2Ω B
(a) (b)

Find RTh (Fig.21b) 4A


RTh = (20 || 5) + 3 + 2 = 9 Ω
A
Find VTh(Fig.21c)
20 Ω 3Ω
5Ω
VTh = −3 × 4 + 5 I 30V I
2Ω B
 30 
VTh = −3 × 4 + 5 
 20 + 5  (c)
VTh = −6 Volts B
RTh
So Its Thevenin’s equivalent circuit is
shown in fig.21d, It is to be noted here VTh
that terminal B is +Ve and A is –Ve.
A
(d) Thevenin’s Equivalent
Circuit

12 Prepared by: Nafees Ahmed www.eedofdit.webnode.com


Norton’s theorem: Fig.21
“Any linear two terminal circuits can be replaced by an
equivalent network consisting of a current source (IN) in parallel with a resistance (RN).”

A A
Any Linear IL
IL
Two
RL ⇒ IN RN RL
terminals
Network
B B
(b) Norton’s Equivalent
(a) Circuit
Fig.22
Where
IN = Isc = Short circuit current at load terminals
RN = Equivalent resistance of the network at load terminals when the
sources are made in-operative (=RTh).
RN
And IL = IN
R N + RL

Example 12: Solve example 5 by Norton’s Theorem.


Solution:
Find RN (Fig.22a) 2
Same as RTh RTh = (2 || 2) = 1Ω 2 IN = Isc
Find IN (Fig.22b) 5A
10V
10
I N = 5A + A
2 (a)
I N = 10 Amps A
IL
RN IN RN RL
IL = IN
R N + RL
1 B
IL = 10 = 2 Amps
1+ 4 (b) Norton’s Equivalent
Circuit
Fig.23

Example 13: Find current and voltage across 5 ohm resistance by Norton’s theorem
(Fig.24a).
Solution:

4Ω 3Ω 4Ω 3Ω
2Ω 5Ω 2Ω RN
6A
15V

(a) (b)

13 Prepared by: Nafees Ahmed www.eedofdit.webnode.com


Find RN (Fig.24b) V1
13 4Ω 3Ω
R N = (4 || 2) + 3 = Ω
3 2Ω 6A
Find IN (Fig.24c) IN
15V
Apply KCL at node V1
 V1 − 15 − 0   V1 − 0  V − 0 
 + −6+ 1 =0
 4   2   3  (c) A
117 IL
V1 = Volts
13 IN RN RL
V − 0 
IN =  1  = 3Amps
 3  B
(13 / 3) 13 (d) Norton’s Equivalent
IL = 3 = Amps
(13 / 3) + 5 9 Circuit
13 Fig.24
VL = I L RL = Volts
3

Example 14: Find Norton’s equivalent circuit at A-B of example 11.


Solution: V1 12 V
Find RN (Fig.21b)
RN = RTh = (20 || 5) + 3 + 2 = 9 Ω 20 Ω 3Ω
5Ω IN
Find IN (Fig.25a) 30V
Apply Node analysis at node V1 after applying 2Ω
source transformation
V=0
 V1 − 30 − 0   V1 − 0   V 1 − 12 − 0 
 +
  +
   = 0 (a)
 20   5   3  A
76
V1 = V
3 IN RN
 V − 12 − 0  2
IN =  1 = A
 20  3 B
Fig.25 (b) Norton’s Equivalent
Circuit

Example 15: Use Norton’s theorem to find out current in 6 ohm resistance and verify it
with Thevenin’s theorem (Fig.26a).
Solution:

4 4 4 I3
A
4 12 V 4 4 12V
2A 2 6 2 2A 2
I1 I2
B
(a) (b) (c)

14 Prepared by: Nafees Ahmed www.eedofdit.webnode.com


By Norton’s Theorem:
Find RN (fig.26b) 4
RN=2 Ohms
A
Find IN (fig 26c) 4 12V
Clearly from mesh 1 2A 2
I1 = 2 A I1 B
Apply KVL in mesh 2
2(I2+I1)-12=0 (d)
So I2 = IN = 4 Amp
2 Fig.26
IL = 4 = 1 Amps
2+6
By Thevenin’s Theorem:
Find RTh (fig.26b)
RTh = RN=2 Ohms
Find VTh (fig 26d)
Clearly from mesh 1
I1 = 2 A
So VTh = 12-2I1= 8 Volts
8
IL = = 1 Amps Hence verified
2+6

Maximum Power Transfer Theorem:


“Maximum power is transferred by a circuit to a
load resistance (RL), when RL is equal to Thevenin’s equivalent resistance (RTh) of that
network.”
A
So for maximum power
RL = RTh RTh IL
And maximum power will be RL
2 VTh
V
Pmax = Th
4 RL B
Fig.27 Thevenin’s Equivalent
Proof: Circuit
Load current will be
VTh
IL = − − − − − (1)
RTh + R L
Power
P = I L2 RL
2
 VTh 
=   RL
 RTh + RL 
RL
= VTh2 − − − − − ( 2)
( RTh + RL ) 2
Differentiating equation equa (2) w. r. t. RL and put dP/dRL = 0

15 Prepared by: Nafees Ahmed www.eedofdit.webnode.com


dP  ( R + RL ) 2 × 1 − RL × ( RTh + RL ) 
= VTh2  Th  − − − − − (3)
dRL  ( RTh + RL ) 4 
RL-RTh = 0
Or RL = RTh Put this in equation (2)
2
V
Pmax = Th
4 RL

Example 16: Find out the value of R for maximum power transfer to this load and find
out the value of maximum power (fig.28a).
Solution:
6V
6 6
2A 15 3 R 15 3 RTh
8V
(a) (b)

Find the RTh (Fig.28b) 6V


21 6 +
RTh = (15 + 6) || 3 = Ω 2A 15 3 VTh
8 I1 I2
Find VTh (Fig.28c) 8V -

(c)
Clearly I1 = 2 A
By KVL in mesh 2
+ 6 I 2 + 6 + 3I 2 + 15( I 2 − I1 ) = 0 Fig.28
So I2 = 1 A
VTh = 3I 2 + 8 = 11Volts
Hence for maximum power transfer to R
21 V2
RL = RTh = Ω and Pmax = Th = 11.524Watt
8 4 RL
Star-Delta and Delta-Star transformation:

1
1
R1 R31 R12

R3 R2
3 2
R23
3 2
(b) Delta ( ∆ ) Connection
(a) Star or WYE (Y)
Connection
Fig.29

16 Prepared by: Nafees Ahmed www.eedofdit.webnode.com


Star to Delta Delta to Star
R R + R2 R3 + R3 R1 R12 R31
R12 = 1 2 R1 =
R3 R12 + R23 + R31
R1 R2 + R2 R3 + R3 R1 R12 R23
R23 = R2 =
R1 R12 + R23 + R31
R1 R2 + R2 R3 + R3 R1 R23 R31
R31 = R3 =
R2 R12 + R23 + R31

Proof:
(Equivalent resistance at 1 - 2)Y = (Equivalent resistance at 1 - 2 )∆
R1 + R2 = (R23 + R31 ) || R12
R12 (R23 + R31 )
R1 + R2 = − − − − − (1)
R12 + R23 + R31
Similarly
(Equivalent resistance at 2 - 3)Y = (Equivalent resistance at 2 - 3)∆
R2 + R3 = (R12 + R31 ) || R23
R (R + R31 )
R2 + R3 = 23 12 − − − − − ( 2)
R12 + R23 + R31
(Equivalent resistance at 3 - 1)Y = (Equivalent resistance at 3 - 1)∆
R3 + R1 = (R12 + R23 ) || R31
R (R + R23 )
R3 + R1 = 31 12 − − − − − (3)
R12 + R23 + R31
Delta to star
Equation (1) + (2) + (3)

2(R12 R23 + R23 R31 + R31 R12 )


2(R1 + R2 + R3 ) =
R12 + R23 + R31
R R + R23 R31 + R31 R12
R1 + R2 + R3 = 12 23 − − − − − ( 4)
R12 + R23 + R31
R12 R31
(4) - (2) R1 = − − − − − (5)
R12 + R23 + R31
R12 R23
(4) - (3) R2 = − − − − − ( 6)
R12 + R23 + R31
R23 R31
(4) – (1) R3 = − − − − − (7 )
R12 + R23 + R31

17 Prepared by: Nafees Ahmed www.eedofdit.webnode.com


Star to Delta
From Equation (5), (6) & (7)
R12 R23 R31 (R12 + R23 + R31 )
R1 R2 + R2 R3 + R3 R1 =
(R12 + R23 + R31 )2
R12 R23 R31
R1 R2 + R2 R3 + R3 R1 = − − − − − (8)
(R12 + R23 + R31 )
R R + R2 R3 + R3 R1
(8) / (7) R12 = 1 2
R3
R R + R2 R3 + R3 R1
(8) / (5) R23 = 1 2
R1
R R + R2 R3 + R3 R1
(8) / (6) R31 = 1 2
R2
Example 17: Find the equivalent resistance between the terminals a-b of the bridge
circuit of the fig.30a
Solution: a
R1=2
a 4 6
2
⇒ R3=2/32 R =1

b 10 14 b 10 14
(a) (b)

Fig. 30
Apply delta to star transformation
R12 R31 6× 4
R1 = = = 2Ω
R12 + R23 + R31 12
R12 R23 6× 2
R2 = = = 1Ω
R12 + R23 + R31 12
R23 R31 4× 2 2
R3 = = = Ω
R12 + R23 + R31 12 3
 2 
Rab = 2 + (1 + 14 ) ||  + 10 
 3 
Rab = 8.234Ω

18 Prepared by: Nafees Ahmed www.eedofdit.webnode.com

Potrebbero piacerti anche